Line integral and continuous gradient

Click For Summary
The discussion focuses on calculating the line integral of the gradient of a function with a continuous gradient over a specified parametric curve. The problem involves evaluating the function at its endpoints, derived from a provided table of values. The correct approach is to use the fundamental theorem of gradients, which states that the integral can be computed as the difference between the function values at the endpoints. The endpoints, calculated from the parametric equations, are (1, 0) and (2, 2), corresponding to function values of 4 and 10, respectively. The final answer for the line integral is 6, confirming the method used is appropriate.
nlsherrill
Messages
320
Reaction score
1

Homework Statement



A table of values of a function f with continuous gradient is given. Find the line integral over C of "gradient F dr" where C has parametric equations x = t2 + 1, y = t3 + t, 0<=t<= 1.


Sorry, don't know latex.

But here's a picture of the table and values
photo.jpg


Homework Equations





The Attempt at a Solution



I'm not even sure what the problem wants me to do with the table. I "assume" since the problem says the table represents values of a function, that I should look at the table and construct a function from it. Well I have tried a few different ways and none of them can stay consistent for more than a line. I feel like this problem is easier than it seems.
 
Physics news on Phys.org
HINT: What does the fundamental theorem of gradients tell you?
 
gabbagabbahey said:
HINT: What does the fundamental theorem of gradients tell you?

So basically with that definition and the table provided, they were looking for you to just evaluate the function at its endpoints correct? I got the right answer, which is 6, but I don't know if what I did was the right way to do it. The parameter t ranges from 0 to 1, so I just plugged in 0 to the x and y components to get (1,0), which on the table=4. Then plugging in 1 for t in the components I got (2,2), which on the table =10. Subtracting the endpoint from the starting point gave me 6. Is this the correct way to do it?

And thank you for your help so far.
 
Yes, the as long as f(x,y)dx+ g(x,yy)dy is an "exact differential", that is, there exist F(x,y) such that dF= f(x)dx+ g(y)dy, then
\int_{t_0}^{t_1} f(x)dx+ g(y)dy= F(x(t_1),y(t_1))- F(x(t_0), y(t_0))

Here, you are given that the F whose values are tabulated gives the required gradient. Since the path is apparently given by x = t^2 + 1, y = t^3 + t, 0\le t\le 1, the endpoints are at (1, 0) and (2, 2).

Your integral is just F(2, 2)- F(1, 0) both of which can be read off the table.
 
Question: A clock's minute hand has length 4 and its hour hand has length 3. What is the distance between the tips at the moment when it is increasing most rapidly?(Putnam Exam Question) Answer: Making assumption that both the hands moves at constant angular velocities, the answer is ## \sqrt{7} .## But don't you think this assumption is somewhat doubtful and wrong?

Similar threads

  • · Replies 8 ·
Replies
8
Views
2K
Replies
3
Views
2K
Replies
2
Views
1K
Replies
6
Views
3K
  • · Replies 2 ·
Replies
2
Views
2K
  • · Replies 5 ·
Replies
5
Views
2K
Replies
12
Views
4K
  • · Replies 5 ·
Replies
5
Views
1K
  • · Replies 22 ·
Replies
22
Views
4K
  • · Replies 11 ·
Replies
11
Views
2K